Last visit was: 03 May 2024, 14:13 It is currently 03 May 2024, 14:13

Close
GMAT Club Daily Prep
Thank you for using the timer - this advanced tool can estimate your performance and suggest more practice questions. We have subscribed you to Daily Prep Questions via email.

Customized
for You

we will pick new questions that match your level based on your Timer History

Track
Your Progress

every week, we’ll send you an estimated GMAT score based on your performance

Practice
Pays

we will pick new questions that match your level based on your Timer History
Not interested in getting valuable practice questions and articles delivered to your email? No problem, unsubscribe here.
Close
Request Expert Reply
Confirm Cancel
SORT BY:
Kudos
Tags:
Show Tags
Hide Tags
VP
VP
Joined: 30 Jan 2016
Posts: 1232
Own Kudos [?]: 4582 [39]
Given Kudos: 128
Send PM
Most Helpful Reply
Senior Manager
Senior Manager
Joined: 29 Mar 2011
Posts: 283
Own Kudos [?]: 339 [6]
Given Kudos: 16
Location: India
Concentration: Strategy, Finance
GMAT 1: 750 Q50 V40
WE:Management Consulting (Consulting)
Send PM
GMAT Club Verbal Expert
Joined: 13 Aug 2009
Status: GMAT/GRE/LSAT tutors
Posts: 6925
Own Kudos [?]: 63756 [5]
Given Kudos: 1775
Location: United States (CO)
GMAT 1: 780 Q51 V46
GMAT 2: 800 Q51 V51
GRE 1: Q170 V170

GRE 2: Q170 V170
Send PM
General Discussion
Tutor
Joined: 16 Oct 2010
Posts: 14869
Own Kudos [?]: 65069 [1]
Given Kudos: 431
Location: Pune, India
Send PM
Re: From time to time there is a public outcry against predatory pricing [#permalink]
1
Bookmarks
Expert Reply
Akela wrote:
From time to time there is a public outcry against predatory pricing- where a company deliberately sells its products at prices low enough to drive its competitors out of business. But this practice clearly should be acceptable, because even after its competitors go out of business, the mere threat of renewed competition will prevent the company from raising its prices to unreasonable levels.

Which one of the following is an assumption on which the argument depends?

(A) Any company that is successful will inevitably induce competitors to enter the market.
(B) It is unlikely that several competing companies will engage in predatory pricing simultaneously.
(C) Only the largest and wealthiest companies can engage in predatory pricing for a sustained period of time.
(D) It is only competition or the threat of competition that keeps companies from raising prices.
(E) Any pricing practice that does not result in unreasonable prices should be acceptable.

Source: LSAT


Premises:
Public is against predatory pricing
But even after competitors go out of business, threat of renewed competition will prevent unreasonable prices.

Conclusion: So this practice should be acceptable

We need an assumption here. An assumption is a missing necessary premise. Let's look for the missing link. The premises talk about how the policy prevents unreasonable prices. The conclusion drawn is that the policy should be acceptable. We are not given the link between unreasonable prices and acceptability.

Option (E) gives us that link: If a practice does not result in unreasonable prices, it should be acceptable.
So (E) is the answer.

(D) It is only competition or the threat of competition that keeps companies from raising prices.
The argument doesn't make this assumption. The argument tells us that threat of competition is sufficient to keep prices reasonable. Is it the only factor? May or may not be. It doesn't change anything in our argument. We are given that it is a factor that keeps prices reasonable. We are told to believe that hence this practice should be acceptable. We are assuming that reasonable prices should make the practice acceptable. What keeps prices reasonable is not the missing link.
CR Forum Moderator
Joined: 25 Jan 2022
Posts: 832
Own Kudos [?]: 643 [1]
Given Kudos: 558
Location: Italy
GPA: 3.8
Send PM
Re: From time to time there is a public outcry against predatory pricing [#permalink]
1
Kudos
Rickooreo wrote:
The conclusion is: But this practice clearly should be acceptable,
Premise (Not sure if this can be considered as a premise or a part of the conclusion, request you to confirm): Because even after its competitors go out of business, the mere threat of renewed competition will prevent the company from raising its prices to unreasonable levels.

Option D, exactly says this, the author thinks renewed competition and that no other factors are involved. Think of this as modified version of A causes B, strengthen will be C does not cause B.
Hence I marked D, can you please help me with my mistake


The author concludes that the practice should be acceptable, because it prevents companies from raising prices to unreasonable levels, because of the threat of competition, even after they have gone out of business as a result of this pricing practice.

D makes the claim that it is "only" the threat of competition that prevents companies from raising prices. If you negate this (its not only the threat of competition or competition that prevents companies from raising prices), the argument is still valid. Competition, or the threat of, could still play some role in preventing the companies from raising prices, so it is still acceptable.

If E is negated (Not all pricing practices that does not result in unreasonable prices should be acceptable) the argument falls apart. The author has stated, that because the practice prevents companies from raising prices to unreasonable levels, it is acceptable. Therefore, E is a required assumption, because if not all practices that does not result in unreasonable are acceptable, then the argument falls apart.
Intern
Intern
Joined: 20 Jan 2018
Posts: 12
Own Kudos [?]: 6 [0]
Given Kudos: 58
Location: India
Schools: ISB '20 (A)
GMAT 1: 620 Q50 V24
GMAT 2: 710 Q51 V34
GPA: 3.9
Send PM
Re: From time to time there is a public outcry against predatory pricing [#permalink]
Akela wrote:
From time to time there is a public outcry against predatory pricing- where a company deliberately sells its products at prices low enough to drive its competitors out of business. But this practice clearly should be acceptable, because even after its competitors go out of business, the mere threat of renewed competition will prevent the company from raising its prices to unreasonable levels.

Which one of the following is an assumption on which the argument depends?

(A) Any company that is successful will inevitably induce competitors to enter the market.
(B) It is unlikely that several competing companies will engage in predatory pricing simultaneously.
(C) Only the largest and wealthiest companies can engage in predatory pricing for a sustained period of time.
(D) It is only competition or the threat of competition that keeps companies from raising prices.
(E) Any pricing practice that does not result in unreasonable prices should be acceptable.

Source: LSAT


I did not get how E is the OA. Experts please help.
Intern
Intern
Joined: 07 Feb 2017
Posts: 34
Own Kudos [?]: 23 [0]
Given Kudos: 51
Send PM
Re: From time to time there is a public outcry against predatory pricing [#permalink]
Akela wrote:
From time to time there is a public outcry against predatory pricing- where a company deliberately sells its products at prices low enough to drive its competitors out of business. But this practice clearly should be acceptable, because even after its competitors go out of business, the mere threat of renewed competition will prevent the company from raising its prices to unreasonable levels.

Which one of the following is an assumption on which the argument depends?

(A) Any company that is successful will inevitably induce competitors to enter the market.
(B) It is unlikely that several competing companies will engage in predatory pricing simultaneously.
(C) Only the largest and wealthiest companies can engage in predatory pricing for a sustained period of time.
(D) It is only competition or the threat of competition that keeps companies from raising prices.
(E) Any pricing practice that does not result in unreasonable prices should be acceptable.

Source: LSAT


Can anyone provide OE for this qn ? OA seems generic.

Premise : Predatory pricing practice is followed by a company to drive competition out of market
Conclusion : Company might continue practicing this pricing due to fear of new competition

Assumption : Even if a company gains monopoly, if there is a price increase, then there will definitely be competition. Hence companies will not raise prices.

A gets its right.

D is a perfect trap. What if the government puts an upper limit on price. So even if there are no competition, the company can't still increase price.
Manager
Manager
Joined: 27 Jul 2016
Posts: 56
Own Kudos [?]: 15 [0]
Given Kudos: 55
GMAT 1: 730 Q49 V40
WE:Consulting (Consulting)
Send PM
Re: From time to time there is a public outcry against predatory pricing [#permalink]
Akela wrote:

(A) Any company that is successful will inevitably induce competitors to enter the market.
(B) It is unlikely that several competing companies will engage in predatory pricing simultaneously.
(C) Only the largest and wealthiest companies can engage in predatory pricing for a sustained period of time.
(D) It is only competition or the threat of competition that keeps companies from raising prices.
(E) Any pricing practice that does not result in unreasonable prices should be acceptable.

Source: LSAT


A - Being successful in implementing predatory pricing does not indicate that a company is successful generally.
B - Not relevant
C - Not relevant
D - I think this is wrong because the passage already states that predatory pricing causes lower price to force competitors out.
E - Only one left
VP
VP
Joined: 13 Apr 2013
Status:It's near - I can see.
Posts: 1479
Own Kudos [?]: 1608 [0]
Given Kudos: 1002
Location: India
Concentration: International Business, Operations
GPA: 3.01
WE:Engineering (Real Estate)
Send PM
Re: From time to time there is a public outcry against predatory pricing [#permalink]
Akela wrote:
From time to time there is a public outcry against predatory pricing- where a company deliberately sells its products at prices low enough to drive its competitors out of business. But this practice clearly should be acceptable, because even after its competitors go out of business, the mere threat of renewed competition will prevent the company from raising its prices to unreasonable levels.

Which one of the following is an assumption on which the argument depends?

(A) Any company that is successful will inevitably induce competitors to enter the market.
(B) It is unlikely that several competing companies will engage in predatory pricing simultaneously.
(C) Only the largest and wealthiest companies can engage in predatory pricing for a sustained period of time.
(D) It is only competition or the threat of competition that keeps companies from raising prices.
(E) Any pricing practice that does not result in unreasonable prices should be acceptable.

Source: LSAT


Though, I got it right, but it was difficult to cross out D. I only crossed out D for its extreme language.

I chose E because it connects conclusion and premise.

GMATNinja , KarishmaB

Please help with POE.
Manager
Manager
Joined: 17 Jan 2017
Posts: 217
Own Kudos [?]: 265 [0]
Given Kudos: 144
Location: India
GPA: 4
WE:Information Technology (Computer Software)
Send PM
Re: From time to time there is a public outcry against predatory pricing [#permalink]
A,B can be eliminated for various reasons.

C, D can be eliminated as they use the extreme word 'Only'

E - This is the best option left out and this needs to be true as it is stated in the argument "But this practice clearly should be acceptable"

Posted from my mobile device
Current Student
Joined: 31 Jul 2017
Status:He came. He saw. He conquered. -- Going to Business School -- Corruptus in Extremis
Posts: 1734
Own Kudos [?]: 5755 [0]
Given Kudos: 3063
Location: United States (MA)
Concentration: Finance, Economics
Send PM
Re: From time to time there is a public outcry against predatory pricing [#permalink]
Expert Reply
Bumping for discussion. Really good assumption LR question from the LSAT!
Manager
Manager
Joined: 09 May 2018
Posts: 98
Own Kudos [?]: 75 [0]
Given Kudos: 75
Send PM
Re: From time to time there is a public outcry against predatory pricing [#permalink]
Akela wrote:
From time to time there is a public outcry against predatory pricing- where a company deliberately sells its products at prices low enough to drive its competitors out of business. But this practice clearly should be acceptable, because even after its competitors go out of business, the mere threat of renewed competition will prevent the company from raising its prices to unreasonable levels.

Which one of the following is an assumption on which the argument depends?

(A) Any company that is successful will inevitably induce competitors to enter the market.
(B) It is unlikely that several competing companies will engage in predatory pricing simultaneously.
(C) Only the largest and wealthiest companies can engage in predatory pricing for a sustained period of time.
(D) It is only competition or the threat of competition that keeps companies from raising prices.
(E) Any pricing practice that does not result in unreasonable prices should be acceptable.

Source: LSAT


Conclusion - The practise should be acceptable.
Pre-thinking assumption that I took - The company will keep its prices low for the threat of renewed competition so this is the only way that company knows how to keep the competitors at bay.
I analysed the options and none of them matched but only D and E seems the contenders.
So, i re-analysed them.
D) We cant make this a necessary condition that this is the only thing that keeps the companies from raising prices but this can be one of the reason and a sufficient reason.
E) Any pricing practice that does not result in unreasonable prices should be acceptable. If I negate this then our conclusion will not hold true.
Senior Manager
Senior Manager
Joined: 24 Dec 2021
Posts: 316
Own Kudos [?]: 24 [0]
Given Kudos: 240
Location: India
Concentration: Finance, General Management
GMAT 1: 690 Q48 V35
GPA: 3.95
WE:Real Estate (Consulting)
Send PM
Re: From time to time there is a public outcry against predatory pricing [#permalink]
The conclusion is: But this practice clearly should be acceptable,
Premise (Not sure if this can be considered as a premise or a part of the conclusion, request you to confirm): Because even after its competitors go out of business, the mere threat of renewed competition will prevent the company from raising its prices to unreasonable levels.

Option D, exactly says this, the author thinks renewed competition and that no other factors are involved. Think of this as modified version of A causes B, strengthen will be C does not cause B.
Hence I marked D, can you please help me with my mistake
CEO
CEO
Joined: 07 Mar 2019
Posts: 2562
Own Kudos [?]: 1822 [0]
Given Kudos: 763
Location: India
WE:Sales (Energy and Utilities)
Send PM
Re: From time to time there is a public outcry against predatory pricing [#permalink]
From time to time there is a public outcry against predatory pricing- where a company deliberately sells its products at prices low enough to drive its competitors out of business. But this practice clearly should be acceptable, because even after its competitors go out of business, the mere threat of renewed competition will prevent the company from raising its prices to unreasonable levels.

Which one of the following is an assumption on which the argument depends?

(A) Any company that is successful will inevitably induce competitors to enter the market. - WRONG. Not necessarily.
(B) It is unlikely that several competing companies will engage in predatory pricing simultaneously. - WRONG. Like A, this is also not necessary.
(C) Only the largest and wealthiest companies can engage in predatory pricing for a sustained period of time. - WRONG. Like A and B, this is also not true necessarily. All the three choices don't touch the reasoning aspect at all.
(D) It is only competition or the threat of competition that keeps companies from raising prices. - WRONG. A likely candidate and it makes fall for itself for simply the use of word 'mere' in passage that might parallel with 'only' used in this choice.
(E) Any pricing practice that does not result in unreasonable prices should be acceptable. - CORRECT. If it's not the case then we have trouble.

The highlighted text is reasoning used by author to conclude that predatory pricing practice should be acceptable.

Answer E.
User avatar
Non-Human User
Joined: 01 Oct 2013
Posts: 17257
Own Kudos [?]: 848 [0]
Given Kudos: 0
Send PM
Re: From time to time there is a public outcry against predatory pricing [#permalink]
Hello from the GMAT Club VerbalBot!

Thanks to another GMAT Club member, I have just discovered this valuable topic, yet it had no discussion for over a year. I am now bumping it up - doing my job. I think you may find it valuable (esp those replies with Kudos).

Want to see all other topics I dig out? Follow me (click follow button on profile). You will receive a summary of all topics I bump in your profile area as well as via email.
GMAT Club Bot
Re: From time to time there is a public outcry against predatory pricing [#permalink]
Moderators:
GMAT Club Verbal Expert
6925 posts
GMAT Club Verbal Expert
238 posts
CR Forum Moderator
832 posts

Powered by phpBB © phpBB Group | Emoji artwork provided by EmojiOne